0 of 15 Questions completed
Questions:
You have already completed the quiz before. Hence you can not start it again.
Quiz is loading…
You must sign in or sign up to start the quiz.
You must first complete the following:
0 of 15 Questions answered correctly
Your time:
Time has elapsed
You have reached 0 of 0 point(s), (0)
Earned Point(s): 0 of 0, (0)
0 Essay(s) Pending (Possible Point(s): 0)
Average score |
|
Your score |
|
A 75-year-old woman with HTN presents with sudden memory deficits and a change in emotional regulation. She is found to have a lacunar infarct, and you feel a disconnection of the Papez circuit may explain her symptoms. Which of the following is not considered a part of the Papez circuit?
A 24-year-old college student presents with uncontrolled epilepsy. His seizures consist of an aura of déjà vu followed by oral automatisms and periods of unresponsiveness. The neurological examination is normal. MRI of the brain is performed and shown below. What is the etiology of this patient’s epilepsy?
Primary generalized dystonia is a genetic disorder that can be seen secondary to a mutation of which of the following genes?
Mild cases of carpal tunnel syndrome will have which of the following findings on electromyography (EMG)?
Match the phase of the Valsalva maneuver with its associated physiology.
A 33-year-old industrial worker presents to the ED complaining of sudden onset nausea, bilateral headache, dyspnea, confusion, and eye irritation. His skin is flushed on physical exam. He states he forgot to wear protective equipment (mask, gloves) while working. A T2 flair MRI of the brain is shown. Exposure to which of the following substances is likely?
A previously healthy 25-year-old female presents to the emergency room with lower extremity weakness, numbness, and urinary retention. Lower extremity hyperreflexia and a sensory level at T10 were appreciated on examination. A contrasted cervical/thoracic and lumbar MRI was performed and shown below. Lumbar puncture was performed after imaging was completed and showed an elevated protein concentration, normal glucose, 3 WBCs, and oligoclonal bands. Rheumatologic studies are negative. Based on the most likely diagnosis, what is the most appropriate next step in medical management?
What is the pathophysiology of the lesion provided below?
A 45-year-old man with a past medical history of type 2 diabetes and COPD presents with a 1-week history of fever, headache, and dizziness. Additionally, for the past two days, he has had intractable vomiting. MRI of the brain with and without contrast is shown below. What is the most appropriate treatment for this patient?
Which of the following is the relay center for hearing?
A 32-year-old Chinese male is referred to the epilepsy clinic after experiencing two episodes concerning for possible seizures. The first episode was unwitnessed, but the patient states that there was an abrupt loss of consciousness and that he woke up with the side of his tongue bitten and generalized myalgias. The second event was witnessed by his wife who noticed one day that the patient was unresponsive and lip-smacking for 2 minutes before returning back to baseline. MRI brain was performed and showed left-sided mesial temporal sclerosis. Which of the following medications should be avoided for this patient?
What is the mechanism of action of abciximab?
A 66-year-old male presents to the emergency room as an unrestrained driver in a motor vehicle accident. While initially only complaining of mild neck pain, within 30 minutes of the collision he started to experience bilateral numbness in his hands and significant symmetric upper limb weakness. Soon thereafter, the weakness extended down to his lower extremities. Neurologic examination is remarkable for loss of pain and temperature sensation in his bilateral upper extremities and symmetric bilateral upper extremity greater than lower extremity weakness. Which of the following is the most likely diagnosis?
Which of the following findings would most likely be found on histopathological analysis of the lesion shown in the image below?
Which of the following medications, when taken alongside statin therapy, increases the risk of statin-induced myopathy?